Search found 90 matches


Answer should be c as we can get to a solution using both the statements; but because the answers are in decimal which cannot be, I think the question is wrong. Had 450 been 470, we would get 30 as answer.
Expert help needed.

by Mom4MBA

Wed Sep 21, 2011 8:50 pm
Forum: Data Sufficiency
Topic: a data from gmat forum
Replies: 5
Views: 1666

Hi pemdas, p/(q^2)=? st(1) p/(q^2)=c where c is a positive integer. Not Sufficient as c can be anything; st(2) q/p=b where b is a positive integer and p=bq . Not Sufficient as p/(q^2)=bq/(q^2)=b/q and b can be any value combined st(1&2): b/q=c, q=b/c --> from st(2) q=bp hence bp=b/c, p=1/c; from...

by Mom4MBA

Tue Jul 12, 2011 9:00 pm
Forum: Data Sufficiency
Topic: Number Prop
Replies: 3
Views: 1050

given w/c = 5/2

statement 1 :
c/m = 5/11
this gives the ration but not actual number
alone not sufficient

statement 2 :
w<30
alone not sufficient

taking both:

w/c = 5/2
c/m = 5/11

so
w/m = 25/22

given w<30

so w=25 hence m=22

both together sufficient

by Mom4MBA

Mon Jul 11, 2011 9:59 pm
Forum: Data Sufficiency
Topic: GMATprep - DS
Replies: 3
Views: 1159

Average of quaterly revenues for 2002

(13+15+16+16)/ 4 = 60/4 = 15

Average of quaterly revenues for 2003

(17+18+17+20)/ 4 = 72/4 = 18

percentage change from 2002 to 2003

[(18-15)/15 ] * 100 = 20 %

by Mom4MBA

Mon Jul 11, 2011 9:39 pm
Forum: Data Sufficiency
Topic: Mean
Replies: 6
Views: 5723
by Mom4MBA

Mon Jul 11, 2011 9:25 pm
Forum: Data Sufficiency
Topic: Mean - Question
Replies: 3
Views: 1285

In the rectangular coordinate system,are the points (r,s) and (u,v) equidistant from the origin? (1) r+s=1 (2) u=1-r;v=1-s; for equidistance √(r² + s²) = √(u² + s²) statement 1: not sufficient as nothing is known about u and v statement 2: √(r² + s²) = √(u² + s²) √(r² + s²) = âˆ...

by Mom4MBA

Mon Jul 11, 2011 9:16 pm
Forum: Data Sufficiency
Topic: Coordinate Geometry!
Replies: 2
Views: 1028

statement 1: alone is not sufficient, as nothing is known about the slopes. infinite lines with various slopes can pass through point (5,1) statement 2: alone is not sufficient. nothing can be done with intercepts alone. taking together plug (5,1) in the equation of both the lines y = mx + c 5 = m1 ...

by Mom4MBA

Mon Jul 11, 2011 9:38 am
Forum: Data Sufficiency
Topic: Is the slope of N less than P?
Replies: 4
Views: 1410

OG 12- DS128

A school administrator will assign each student in a group of n students to one of m classrooms. If 3 < m < 13 < n, is it possible to assign each of the n students to one of the m classrooms so that each classroom has the same number of students assigned to it? (1) It is possible to assign each of ...

by Mom4MBA

Sun Jul 10, 2011 9:35 pm
Forum: Data Sufficiency
Topic: OG 12- DS128
Replies: 3
Views: 1173

if the question is reworded that K and y=2x+3 are perpendicular (and hence we know from K's slope the relation m1*m2=-1). Also, which other additional/new info will make B sufficient. you are right, had it been given that the line is perpendicular to y=2x+3, then even statement 2 would have been su...

by Mom4MBA

Thu Jul 07, 2011 1:58 am
Forum: Data Sufficiency
Topic: OG-12.94
Replies: 2
Views: 1298

If v,w,x,y are non-negative integers, and a=3w2v b=5z3y2x is a/b a terminating decimal? (1) w>y (2) v>x so far, only 6 out of 85 people answered correctly people will definitely give you wrong answer if you give them wrong question. you should have mentioned the powers properly. Had you written pro...

by Mom4MBA

Thu Jul 07, 2011 1:48 am
Forum: Data Sufficiency
Topic: Tough Kaplan DS
Replies: 11
Views: 1967

Just a try.

A fraction can be terminating only if the denominator is 2 and/or 5 , else it is recurring.

so none of the statements alone or combined tell us anything about what the denominators x,y,z are hence answer should be (e) none of the statements are sufficient.

by Mom4MBA

Wed Jul 06, 2011 9:06 pm
Forum: Data Sufficiency
Topic: Tough Kaplan DS
Replies: 11
Views: 1967

Let Y take 'a' days to complete 'w' widgets then X will take 'a+2' days to complete 'w' widgets therefore the rate for Y will be w/a widgets per day and the rate for X will be w/(a+2) widgets per day together their rate of completing 'w' widgets will be w/a + w/(a+2) = (2a+2)w/(a(a+2))...........(i)...

by Mom4MBA

Wed Jun 15, 2011 9:36 pm
Forum: Problem Solving
Topic: Quant Rev. #173 PS
Replies: 2
Views: 999

5a + 4 = x
23b + 7 = x

equate:

5a + 4 = 23b + 7
23b - 5a = -3

b can take values 0,1,2,3,4 above 4 x will be > 100, which is not allowed

for b=0, a=3/5 not possible
for b=1, a=26/5 not possible
for b=2, a=49/5 not possible
for b=3, a=72/5 not possible
for b=4, a=95/5=19
so x=99

by Mom4MBA

Wed Jun 15, 2011 4:24 am
Forum: Problem Solving
Topic: Remainder
Replies: 4
Views: 1278

For class A let 'a' be minimum height, so r=g-a ; range = max - min For class B let 'b' be minimum height, so s=h-b statement 1: r < s g-a<h-b but we do not know anything about g and h so insufficient statement 2: g > h r+a > s+b but we do not know anything about r and s so insufficient taking both ...

by Mom4MBA

Wed Jun 15, 2011 4:08 am
Forum: Data Sufficiency
Topic: Are x and y both positive ?
Replies: 2
Views: 1006

How many integers are there between c and d? (1) Neither c nor d is an integer. (2) c - d = 3 Statement 1 alone is not sufficient Statement 2: if both integers then just 2 integers will be between them if both non integers then 3 integers will be between them so statement 2 alone not sufficient hen...

by Mom4MBA

Wed Jun 15, 2011 3:48 am
Forum: Data Sufficiency
Topic: Little confusing!!!
Replies: 2
Views: 912